Need to ask this question again

  • Thread starter Thread starter Jamin2112
  • Start date Start date
Click For Summary
The discussion revolves around the continuity of functions, specifically addressing a homework problem. It clarifies that a function f(x,y) is continuous at a point (x0,y0) if it is defined there, and discontinuous if it is defined but does not meet continuity criteria. Participants agree that if a function is not defined at a point, it is neither continuous nor discontinuous. An example is provided with the function f(x)=sqrt(x), which is only defined for x>=0, illustrating the point that it cannot be classified as continuous or discontinuous at points where it is undefined. The conversation emphasizes the importance of definitions in understanding function continuity.
Jamin2112
Messages
973
Reaction score
12
A few days ago, I asked about the continuity of a function, in regards to a problem on my homework. Something still seems wrong, even though today my professor insisted that the given definitions are correct. Look here: http://www.math.washington.edu/~sullivan/3261_sp10.pdf"

A function f(x,y) is continuous at a point (x0,y0) if f(x,y) is defined at (x0,y0), ...

A function f(x,y) is discontinuous at a point (x0,y0) if f(x,y) is defined at (x0,y0), ...


So if a function isn't defined, then it is neither continuous or discontinuous?
 
Last edited by a moderator:
Physics news on Phys.org
According to those definitions, I'll agree. Yes, if it's not defined it's neither continuous nor discontinuous.
 
Dick said:
According to those definitions, I'll agree. Yes, if it's not defined it's neither continuous nor discontinuous.

Weird
 
Jamin2112 said:
Weird

Well, think about f(x)=sqrt(x). That's only defined for x>=0. You certainly don't want to say it's continuous at x=(-1). And it also sounds a little weird to say it's discontinuous at x=(-1). It's just not defined. Not all that strange.
 
Question: A clock's minute hand has length 4 and its hour hand has length 3. What is the distance between the tips at the moment when it is increasing most rapidly?(Putnam Exam Question) Answer: Making assumption that both the hands moves at constant angular velocities, the answer is ## \sqrt{7} .## But don't you think this assumption is somewhat doubtful and wrong?

Similar threads

  • · Replies 3 ·
Replies
3
Views
2K
  • · Replies 4 ·
Replies
4
Views
2K
  • · Replies 5 ·
Replies
5
Views
1K
  • · Replies 11 ·
Replies
11
Views
3K
  • · Replies 1 ·
Replies
1
Views
4K
Replies
4
Views
4K
  • · Replies 4 ·
Replies
4
Views
2K
Replies
2
Views
2K
Replies
1
Views
2K
  • · Replies 13 ·
Replies
13
Views
3K